← Back to Topics

geriatrics

Practice targeted AMC-style multiple-choice questions on geriatrics.

Related Topics

Right leg affected by warfarin necrosis
Image by Bakoyiannis C, Karaolanis G, Patelis N, Maskanakis A, Tsaples G, Klonaris C, Georgopoulos S, Liakakos T CC BY 4.0 · Source

A 72-year-old female presents with right leg pain and skin changes after starting warfarin for atrial fibrillation. Her INR is 3.5. Examination reveals the findings in the image. What is the MOST appropriate next step in management?

Mark this as a high-quality question
Mark this as a poor-quality or problematic question
Elective abdominal ultrasonography of a 64 year old woman with past colicky pain but no current pain. It shows a mildly dilated common bile duct with what appears to be a gallstone (black arrow). The green and blue dots are blood flow, since the exam is done as a Doppler ultrasound to distinguish the bile duct from blood vessels.
Image by Mikael Häggström CC0 1.0 · Source

A 64-year-old woman presents for follow-up after experiencing intermittent episodes of right upper quadrant colicky pain over the past year, which have since resolved. She denies any current pain, fever, jaundice, or changes in bowel habits. Physical examination is unremarkable, and recent liver function tests are within normal limits. An elective abdominal ultrasound was performed, yielding the image shown. Considering the clinical context and the findings demonstrated, what is the most appropriate next step in her management?

Mark this as a high-quality question
Mark this as a poor-quality or problematic question

A 72-year-old man with a history of hypertension, type 2 diabetes, and chronic kidney disease (CKD) presents to his general practitioner for a routine check-up. His current medications include metformin, insulin glargine, amlodipine, and aspirin. His blood pressure today is 150/90 mmHg. Laboratory results show a serum creatinine of 2.5 mg/dL (baseline 2.0 mg/dL), potassium of 5.4 mEq/L, and a urine albumin-to-creatinine ratio (ACR) of 350 mg/g. The GP decides to add an ACE inhibitor to his medication regimen to help manage his blood pressure and proteinuria. One week later, the patient returns complaining of fatigue and muscle weakness. Repeat laboratory testing reveals a serum creatinine of 3.1 mg/dL and a potassium of 6.2 mEq/L. Which of the following is the most appropriate next step in managing this patient?

Mark this as a high-quality question
Mark this as a poor-quality or problematic question
CT scan showing hemothorax caused by warfarin use
Image by Cevik Y CC BY 3.0 · Source

A 78-year-old male on warfarin for atrial fibrillation presents with acute dyspnea and pleuritic chest pain. His INR is 7.0. A chest CT is performed (image attached). After initial resuscitation, what is the MOST appropriate intervention?

Mark this as a high-quality question
Mark this as a poor-quality or problematic question

A 72-year-old man with a history of chronic kidney disease stage 4, hypertension, and type 2 diabetes presents to the emergency department with confusion and lethargy. His family reports that he has been increasingly drowsy over the past two days. On examination, he is disoriented to time and place, with a blood pressure of 150/90 mmHg, heart rate of 88 bpm, respiratory rate of 20 breaths per minute, and temperature of 36.5°C. Laboratory tests reveal: sodium 130 mmol/L, potassium 5.8 mmol/L, bicarbonate 18 mmol/L, urea 25 mmol/L, creatinine 450 µmol/L, and glucose 8 mmol/L. An ECG shows peaked T waves. What is the most appropriate immediate management step?

Mark this as a high-quality question
Mark this as a poor-quality or problematic question
CT scan showing hemothorax caused by warfarin use
Image by Cevik Y CC BY 3.0 · Source

A 78-year-old male on warfarin presents with dyspnea and pleuritic chest pain after a fall. INR is 7.0. A chest CT is performed (image attached). After initial resuscitation, what is the MOST appropriate next step?

Mark this as a high-quality question
Mark this as a poor-quality or problematic question

A 68-year-old woman presents with a 2-week history of progressive weakness in her right arm and leg, associated with a tremor in the right hand. She also reports feeling more tired than usual. Her general practitioner noted an elevated TSH (12 mIU/L, reference range 0.4-4.0 mIU/L) on routine blood tests done a month ago, but free T4 was within the normal range (15 pmol/L, reference range 10-20 pmol/L). On examination, she has mild right-sided hemiparesis (MRC grade 4/5) and a coarse tremor in the right hand, worse with action. Reflexes are brisk on the right. The rest of the neurological exam is unremarkable. She has mild bradycardia (55 bpm) but no other signs of overt hypothyroidism. What is the most appropriate initial investigation?

Mark this as a high-quality question
Mark this as a poor-quality or problematic question

A 72-year-old man presents to the clinic with progressive shortness of breath and a chronic cough producing white sputum. He has a 40-pack-year smoking history and worked in construction for most of his life. On examination, he has clubbing of the fingers and crackles at the lung bases. A chest X-ray shows diffuse reticular opacities, particularly at the lung bases. What is the most likely diagnosis?

Mark this as a high-quality question
Mark this as a poor-quality or problematic question

A 70-year-old woman with a history of diabetes presents with fever, cough, and weight loss. A chest X-ray shows a cavitary lesion in the right upper lobe. What is the most likely diagnosis?

Mark this as a high-quality question
Mark this as a poor-quality or problematic question
Fluoroscopic image of common bile duct stone seen at the time of ERCP. The stone is impacted in the distal common bile duct.
Image by Samir धर्म at en.wikipedia CC BY-SA 3.0 · Source

A 68-year-old woman presents to the emergency department with a 2-day history of worsening right upper quadrant abdominal pain radiating to her back, associated with fever, chills, and yellowing of her eyes. She denies prior similar episodes but reports occasional post-prandial discomfort. On examination, she is jaundiced and tender in the right upper quadrant. Vital signs are: temperature 38.5°C, heart rate 110 bpm, blood pressure 110/70 mmHg, respiratory rate 20 breaths per minute, and oxygen saturation 97% on room air. Initial laboratory investigations reveal: white blood cell count 14,000/µL, total bilirubin 6.0 mg/dL, alkaline phosphatase 450 U/L, ALT 300 U/L, and AST 250 U/L. An ERCP is performed, and the image is obtained. Following successful intervention to relieve the obstruction, the patient's symptoms begin to improve. Considering the likely underlying cause of this presentation and the findings during the procedure, what is the MOST appropriate next step in managing this patient's condition?

Mark this as a high-quality question
Mark this as a poor-quality or problematic question

A 72-year-old man with a history of type 2 diabetes mellitus, hypertension, and chronic kidney disease (stage 3) presents with a 2-week history of exertional dyspnea and fatigue. He reports no chest pain but has noticed occasional palpitations. On examination, his blood pressure is 150/90 mmHg, heart rate is 110 bpm (irregularly irregular), respiratory rate is 20 breaths per minute, and oxygen saturation is 94% on room air. Cardiovascular examination reveals a variable intensity S1, no S3 or S4, and a soft systolic murmur at the apex. Lung fields are clear. An ECG shows absent P waves and irregularly irregular QRS complexes. Which of the following is the most appropriate next step in management?

Mark this as a high-quality question
Mark this as a poor-quality or problematic question

A 68-year-old woman is admitted to the hospital with confusion and muscle weakness. Her medications include hydrochlorothiazide for hypertension. Initial laboratory results show: Sodium 120 mmol/L (Normal: 135-145 mmol/L), Potassium 3.8 mmol/L (Normal: 3.5-5.0 mmol/L), Chloride 90 mmol/L (Normal: 95-105 mmol/L). What is the most likely cause of her hyponatremia?

Mark this as a high-quality question
Mark this as a poor-quality or problematic question
X-ray of hiatal hernia
Image by Ahmed Farhat, Daryn Towle CC BY 4.0 · Source

An 82-year-old woman is admitted to the hospital with increasing shortness of breath and intermittent chest discomfort over the past week. She reports some difficulty swallowing larger food boluses recently but denies significant heartburn. Her past medical history includes hypertension and osteoarthritis. On examination, her vital signs are stable: BP 135/85 mmHg, HR 78 bpm, RR 18/min, SpO2 94% on room air, Temp 36.8°C. Auscultation of the chest reveals decreased breath sounds at the left base. A central venous catheter is noted in the right subclavian vein. Initial blood tests show a mild normocytic anaemia (Hb 105 g/L) and normal inflammatory markers. An urgent imaging study is performed. Considering the clinical presentation and the provided imaging study, what is the most appropriate next step in the management of this patient?

Mark this as a high-quality question
Mark this as a poor-quality or problematic question

A 72-year-old man with a history of hypertension, type 2 diabetes, and chronic kidney disease (CKD) presents to his general practitioner for a routine check-up. His current medications include metformin, insulin, amlodipine, and aspirin. His blood pressure today is 150/90 mmHg. Laboratory results show a serum creatinine of 2.5 mg/dL (221 micromol/L) and a potassium level of 5.4 mEq/L. The GP decides to add an ACE inhibitor to his treatment regimen to help manage his blood pressure and provide renal protection. One week later, the patient returns complaining of fatigue and muscle weakness. Repeat laboratory tests reveal a serum creatinine of 3.1 mg/dL (274 micromol/L) and a potassium level of 6.2 mEq/L. Which of the following is the most appropriate next step in managing this patient?

Mark this as a high-quality question
Mark this as a poor-quality or problematic question
An incarcerated inguinal hernia as seen on CT
Image by James Heilman, MD CC BY-SA 3.0 · Source

A 78-year-old male presents to the emergency department with a 6-hour history of sudden onset, severe pain and swelling in his right groin. He reports nausea but no vomiting. On examination, there is a tender, firm, non-reducible lump in the right inguinal region. Bowel sounds are present. Vitals are stable: BP 130/80, HR 75, Temp 36.8°C. A CT scan of the pelvis is performed, shown in the image. Based on the clinical presentation and the findings demonstrated in the image, what is the most appropriate immediate next step in the management of this patient?

Mark this as a high-quality question
Mark this as a poor-quality or problematic question
Diverticulitis in the left lower quadrant. There is outpouching of the colonic wall, wall thickening, and surrounding fat stranding.
Image by James Heilman, MD CC BY-SA 3.0 · Source

A 68-year-old woman presents with 3 days of left lower quadrant pain. She reports mild nausea but no vomiting or fever. On examination, she is afebrile, BP 130/80, HR 78, O2 sat 98% on air. There is localised tenderness in the LLQ. Bloods show WCC 13.2, CRP 45. A CT scan is performed. Considering the clinical presentation and the findings demonstrated in the image, what is the most appropriate initial management plan?

Mark this as a high-quality question
Mark this as a poor-quality or problematic question

A 62-year-old woman presents to the emergency department with confusion and lethargy. Her family reports she has been increasingly forgetful over the past week and has had a decreased appetite. She has a history of type 2 diabetes mellitus and hypertension, for which she takes metformin and lisinopril. On examination, she is disoriented to time and place, with dry mucous membranes and decreased skin turgor. Her blood pressure is 100/60 mmHg, heart rate is 110 bpm, and temperature is 37.2°C. Laboratory tests reveal a serum sodium level of 118 mmol/L, serum osmolality of 260 mOsm/kg, and urine osmolality of 500 mOsm/kg. Which of the following is the most appropriate initial management for this patient?

Mark this as a high-quality question
Mark this as a poor-quality or problematic question
Inguinal hernia right view. Adult male with an inguinal hernia, showing bulging in the groin area.
Image by IkeTheSloth CC0 1.0 · Source

A 72-year-old male presents to the clinic complaining of a painless swelling in his right groin that has been present for several years. He reports that the swelling occasionally increases in size with prolonged standing or heavy lifting, but it usually reduces when he lies down. He denies any acute pain, nausea, vomiting, or changes in bowel habits. His past medical history includes well-controlled type 2 diabetes mellitus and a remote appendectomy. On physical examination, his vital signs are within normal limits. Abdominal examination is unremarkable. Examination of the groin reveals the finding shown in the image. The swelling is easily reducible when the patient is supine. Which of the following is the MOST appropriate initial management strategy for this patient?

Mark this as a high-quality question
Mark this as a poor-quality or problematic question
CT scan showing hemothorax caused by warfarin use
Image by Cevik Y CC BY 3.0 · Source

A 68-year-old male on warfarin presents post-fall with dyspnea and pleuritic chest pain. INR is 4.8. Vitals: HR 115, BP 95/60, RR 26, SpO2 89% on air. The provided image was obtained. What is the MOST appropriate next step?

Mark this as a high-quality question
Mark this as a poor-quality or problematic question

A 68-year-old man with atrial fibrillation is scheduled for elective hip replacement surgery. He is currently on warfarin with a target INR of 2.0-3.0. His INR is 2.5 at the preoperative assessment. What is the most appropriate management of his anticoagulation therapy in preparation for surgery?

Mark this as a high-quality question
Mark this as a poor-quality or problematic question
Diverticular disease
Image by James Heilman, MD CC BY-SA 4.0 · Source

A 65-year-old presents with 2 days of left lower quadrant pain, fever (38.5°C), and nausea. On examination, there is localised tenderness in the LLQ. Vitals: BP 130/80, HR 90, RR 16, SpO2 98%. CRP is 150 mg/L, WCC 18 x 10^9/L. This image is obtained as part of the workup. Based on the clinical presentation and imaging findings, what is the most appropriate initial management step?

Mark this as a high-quality question
Mark this as a poor-quality or problematic question
Right leg affected by warfarin necrosis
Image by Bakoyiannis C, Karaolanis G, Patelis N, Maskanakis A, Tsaples G, Klonaris C, Georgopoulos S, Liakakos T CC BY 4.0 · Source

A 65-year-old patient recently started on warfarin for deep vein thrombosis presents with a painful, expanding skin lesion on their leg. They report the lesion appeared suddenly and has worsened over the past 24 hours. INR is 2.8. Considering the clinical presentation and the appearance of the lesion, what is the most appropriate immediate management step?

Mark this as a high-quality question
Mark this as a poor-quality or problematic question
Upright X-ray demonstrating small bowel obstruction
Image by James Heilman, MD CC BY-SA 3.0 · Source

A 72-year-old male presents with vomiting and abdominal pain. An abdominal X-ray is performed. What is the MOST likely underlying cause?

Mark this as a high-quality question
Mark this as a poor-quality or problematic question
Fluoroscopic image of common bile duct stone seen at the time of ERCP. The stone is impacted in the distal common bile duct.
Image by Samir धर्म at en.wikipedia CC BY-SA 3.0 · Source

A 68-year-old woman presented to the emergency department with right upper quadrant abdominal pain, fever, and jaundice. Initial investigations showed a WCC of 14,000, total bilirubin 6.0, ALP 450, ALT 300. She was diagnosed with acute cholangitis and underwent urgent ERCP, during which the image was obtained. Following successful endoscopic stone extraction and significant clinical improvement with normalising inflammatory markers and liver function tests, what is the MOST appropriate next step in the long-term management of this patient's condition?

Mark this as a high-quality question
Mark this as a poor-quality or problematic question
An incarcerated inguinal hernia as seen on CT
Image by James Heilman, MD CC BY-SA 3.0 · Source

A 78-year-old man presents to the emergency department with acute, severe right groin pain and a palpable, non-reducible mass. He reports associated nausea but denies vomiting or changes in bowel habit. His vital signs are within normal limits, and bowel sounds are audible. A CT scan of the pelvis is performed, the findings of which are depicted in the image. Given the clinical context and the information revealed by the imaging study, what is the most appropriate immediate management strategy?

Mark this as a high-quality question
Mark this as a poor-quality or problematic question
Right leg affected by warfarin necrosis
Image by Bakoyiannis C, Karaolanis G, Patelis N, Maskanakis A, Tsaples G, Klonaris C, Georgopoulos S, Liakakos T CC BY 4.0 · Source

An 82-year-old woman with a history of paroxysmal atrial fibrillation and a recent hospitalisation for pneumonia was discharged 5 days ago on warfarin 5mg daily. She presents to the emergency department complaining of increasing pain and a rapidly evolving lesion on her right lower leg over the past 48 hours. She denies trauma. Her vital signs are stable: BP 130/80 mmHg, HR 78 bpm, RR 16/min, Temp 36.8°C. Physical examination reveals the appearance shown in the image on her right anterior lower leg. Her INR today is 4.5. She has no known history of protein C or S deficiency. Considering the patient's history, current medication, laboratory result, and the clinical appearance depicted, which of the following represents the most appropriate immediate management strategy?

Mark this as a high-quality question
Mark this as a poor-quality or problematic question
An incarcerated inguinal hernia as seen on CT
Image by James Heilman, MD CC BY-SA 3.0 · Source

An 80-year-old male presents to the emergency department with sudden onset of severe right groin pain and a non-reducible bulge. He has a history of a reducible inguinal hernia. Vitals are stable. A CT scan of the pelvis is performed. Considering the clinical presentation and the provided imaging, what is the most appropriate immediate management step?

Mark this as a high-quality question
Mark this as a poor-quality or problematic question
Right leg affected by warfarin necrosis
Image by Bakoyiannis C, Karaolanis G, Patelis N, Maskanakis A, Tsaples G, Klonaris C, Georgopoulos S, Liakakos T CC BY 4.0 · Source

A 68-year-old woman recently started on warfarin for a pulmonary embolism presents with a painful, rapidly expanding purpuric lesion on her thigh, which developed two days ago. Her INR is 2.5. She has no history of trauma to the area. Considering the clinical presentation and the appearance of the lesion, what is the most likely underlying pathophysiological process?

Mark this as a high-quality question
Mark this as a poor-quality or problematic question

A 60-year-old woman presents to the clinic with a 2-month history of progressive shortness of breath and a dry cough. She has a history of rheumatoid arthritis and is currently on methotrexate and low-dose prednisone. On examination, she has fine inspiratory crackles at the lung bases. A high-resolution CT scan of the chest shows reticular opacities and honeycombing predominantly in the lower lobes. What is the most likely diagnosis?

Mark this as a high-quality question
Mark this as a poor-quality or problematic question

A 72-year-old man presents to the emergency department with confusion and difficulty walking. His family reports that he has been increasingly forgetful over the past few months and has had several falls. He has a history of hypertension and hyperlipidemia, for which he takes lisinopril and atorvastatin. On examination, he is disoriented to time and place, with a blood pressure of 140/85 mmHg, heart rate of 78 bpm, and a wide-based gait. Neurological examination reveals brisk reflexes and a positive Romberg sign. A CT scan of the head shows mild cortical atrophy but no acute lesions. Which of the following is the most likely diagnosis?

Mark this as a high-quality question
Mark this as a poor-quality or problematic question

A 72-year-old woman is admitted to the hospital with increasing confusion and generalized weakness over the past week. Her medical history includes hypertension, for which she takes hydrochlorothiazide 25mg daily. On examination, she is lethargic but arousable. Her blood pressure is 110/70 mmHg, heart rate is 88 bpm, and respiratory rate is 16 breaths per minute. Neurological examination reveals decreased reflexes. Initial laboratory investigations reveal the following: Sodium 118 mmol/L, Potassium 3.8 mmol/L, Chloride 85 mmol/L, Bicarbonate 24 mmol/L, BUN 15 mg/dL, Creatinine 0.8 mg/dL, Glucose 90 mg/dL. Urine osmolality is 600 mOsm/kg. Serum osmolality is 250 mOsm/kg. Which of the following is the most appropriate initial management strategy?

Mark this as a high-quality question
Mark this as a poor-quality or problematic question

A 72-year-old woman presents with sudden onset shortness of breath and pleuritic chest pain. She had knee surgery 10 days ago. She has a history of hypertension. On examination, she is anxious, respiratory rate is 24, heart rate 105, BP 130/80, SpO2 92% on room air. Lungs are clear. ECG shows sinus tachycardia. There is mild swelling in the operated leg. Considering the likely diagnosis, which initial investigation is most appropriate?

Mark this as a high-quality question
Mark this as a poor-quality or problematic question
CT scan of hepatocellular carcinoma, without and with IV contrast
Image by Zhenyu Pan, Guozi Yang, Tingting Yuan, Lihua Dong, Lihua Dong CC BY 4.0 · Source

A 68-year-old man with known alcoholic cirrhosis presents for routine surveillance. He reports mild fatigue. Liver function tests are within his baseline (Child-Pugh A). Alpha-fetoprotein is 150 ng/mL. The provided images are obtained. Considering the clinical context and the findings demonstrated in the images, which of the following treatment modalities is most likely to be considered first for this patient?

Mark this as a high-quality question
Mark this as a poor-quality or problematic question
X-ray of hiatal hernia
Image by Ahmed Farhat, Daryn Towle CC BY 4.0 · Source

A 78-year-old male presents to the emergency department with a 2-day history of intermittent upper abdominal pain radiating to the chest and mild dysphagia. He denies shortness of breath or fever. Vital signs are stable. A chest X-ray is obtained. Considering the radiographic findings in the context of the patient's presentation, what is the most appropriate immediate management plan?

Mark this as a high-quality question
Mark this as a poor-quality or problematic question
CT scan showing hemothorax caused by warfarin use
Image by Cevik Y CC BY 3.0 · Source

A 78-year-old male on warfarin for atrial fibrillation presents to the emergency department with sudden onset of severe dyspnoea and right-sided pleuritic chest pain. He denies trauma. On examination, he is tachypnoeic, oxygen saturation is 92% on room air. Blood pressure 110/70 mmHg, heart rate 95 bpm. Chest examination reveals decreased breath sounds on the right. A CT scan of the chest is performed. Considering the patient's clinical presentation, medical history, and the findings demonstrated in the provided image, which of the following represents the most appropriate immediate management strategy?

Mark this as a high-quality question
Mark this as a poor-quality or problematic question

A 78-year-old woman with a history of heart failure (reduced ejection fraction), type 2 diabetes, and recent hospitalisation for pneumonia presents to her GP with increasing fatigue and a single episode of dizziness leading to a fall without loss of consciousness. Her current medications are Digoxin 125 mcg daily, Furosemide 40 mg daily, Ramipril 5 mg daily, Metformin 500 mg BD, and Amoxicillin 500 mg TDS (started 5 days ago for pneumonia). An ECG in clinic shows sinus rhythm, rate 70 bpm, and a QTc of 530 ms. An ECG from 6 months prior had a QTc of 440 ms. Recent laboratory tests show Na 138 mmol/L, K 4.2 mmol/L, Mg 0.9 mmol/L, Ca 2.3 mmol/L, Creatinine 90 µmol/L, and Digoxin level 1.1 ng/mL. Considering the clinical context and investigations, which of the following is the most likely significant contributor to the observed QTc prolongation?

Mark this as a high-quality question
Mark this as a poor-quality or problematic question
X-ray of hiatal hernia
Image by Ahmed Farhat, Daryn Towle CC BY 4.0 · Source

An 82-year-old patient presents to the emergency department with a 3-month history of intermittent retrosternal chest discomfort, worse after meals, and mild shortness of breath on exertion. They report occasional regurgitation but deny significant dysphagia. Past medical history includes hypertension and osteoarthritis. Medications are perindopril and paracetamol. On examination, vital signs are stable, and lung sounds are clear. A chest X-ray is performed. Based on the findings in the image, what is the most appropriate next step in management or investigation?

Mark this as a high-quality question
Mark this as a poor-quality or problematic question
X-ray of hiatal hernia
Image by Ahmed Farhat, Daryn Towle CC BY 4.0 · Source

An 82-year-old patient presents to their GP with a 3-month history of intermittent retrosternal chest discomfort, often described as a burning sensation, worse after meals and when lying flat. They also report mild shortness of breath on exertion and occasional regurgitation, but deny significant dysphagia or weight loss. Past medical history includes hypertension and osteoarthritis. Medications are perindopril and paracetamol. On examination, vital signs are stable (BP 130/80, HR 75, RR 16, SpO2 97% on air), and lung sounds are clear. A chest X-ray is performed as part of the workup. Based on the clinical context and the findings demonstrated in the image, what is the most appropriate next step in investigation?

Mark this as a high-quality question
Mark this as a poor-quality or problematic question

A 70-year-old man presents with a vertebral fracture after a minor fall. Reviewing risk factors for osteoporosis, which of the following is the most likely risk factor?

Mark this as a high-quality question
Mark this as a poor-quality or problematic question
X-ray of hiatal hernia
Image by Ahmed Farhat, Daryn Towle CC BY 4.0 · Source

A 72-year-old male presents with increasing shortness of breath and epigastric discomfort, particularly after meals. He has a history of well-controlled hypertension and takes aspirin daily. An X-ray is performed. What is the most appropriate next step in management?

Mark this as a high-quality question
Mark this as a poor-quality or problematic question
Fluoroscopic image of common bile duct stone seen at the time of ERCP. The stone is impacted in the distal common bile duct.
Image by Samir धर्म at en.wikipedia CC BY-SA 3.0 · Source

A 68-year-old woman presents to the emergency department with a 2-day history of right upper quadrant abdominal pain, fever, and yellowing of her eyes. Her vital signs are stable, but laboratory investigations reveal elevated inflammatory markers and liver function tests consistent with biliary obstruction and infection. An ERCP is performed, and the image is obtained. Following successful intervention to relieve the obstruction, the patient's symptoms begin to improve. Considering the likely underlying cause of this presentation and the findings during the procedure, what is the MOST appropriate next step in managing this patient's condition?

Mark this as a high-quality question
Mark this as a poor-quality or problematic question
An incarcerated inguinal hernia as seen on CT
Image by James Heilman, MD CC BY-SA 3.0 · Source

A 70-year-old male presents with a painful groin mass and constipation. The provided image was taken. What is the most appropriate next step?

Mark this as a high-quality question
Mark this as a poor-quality or problematic question
Diverticular disease
Image by James Heilman, MD CC BY-SA 4.0 · Source

A 68-year-old male presents to the ED with left lower quadrant abdominal pain, fever, and leukocytosis. A CT scan of the abdomen and pelvis is performed. Based on the image, what is the most appropriate next step in management?

Mark this as a high-quality question
Mark this as a poor-quality or problematic question
X-ray of hiatal hernia
Image by Ahmed Farhat, Daryn Towle CC BY 4.0 · Source

An 82-year-old woman is admitted to the hospital with increasing shortness of breath and intermittent chest discomfort over the past week. She reports some difficulty swallowing larger food boluses recently but denies significant heartburn. Her past medical history includes hypertension and osteoarthritis. On examination, her vital signs are stable: BP 135/85 mmHg, HR 78 bpm, RR 18/min, SpO2 94% on room air, Temp 36.8°C. Auscultation of the chest reveals decreased breath sounds at the left base. A central venous catheter is noted in the right subclavian vein. Initial blood tests show a mild normocytic anaemia (Hb 105 g/L) and normal inflammatory markers. An urgent imaging study is performed. Considering the clinical presentation and the findings on the imaging study, which of the following is the most likely underlying cause of the patient's anaemia?

Mark this as a high-quality question
Mark this as a poor-quality or problematic question

A 70-year-old man with a history of hypertension and heart failure is being managed with an ACE inhibitor. He presents to the emergency department complaining of dizziness, especially when standing up. His blood pressure is 90/60 mmHg, and his other vital signs are stable. Which of the following is the most appropriate initial step in managing this patient?

Mark this as a high-quality question
Mark this as a poor-quality or problematic question
Right leg affected by warfarin necrosis
Image by Bakoyiannis C, Karaolanis G, Patelis N, Maskanakis A, Tsaples G, Klonaris C, Georgopoulos S, Liakakos T CC BY 4.0 · Source

An 82-year-old woman with a history of paroxysmal atrial fibrillation and a recent hospitalisation for pneumonia was discharged 5 days ago on warfarin 5mg daily. She presents to the emergency department complaining of increasing pain and a rapidly evolving lesion on her right lower leg over the past 48 hours. She denies trauma. Her vital signs are stable: BP 130/80 mmHg, HR 78 bpm, RR 16/min, Temp 36.8°C. Physical examination reveals the appearance shown in the image on her right anterior lower leg. Her INR today is 4.5. She has no known history of protein C or S deficiency. Considering the patient's history, current medication, laboratory result, and the clinical appearance depicted, which of the following represents the most appropriate immediate management strategy?

Mark this as a high-quality question
Mark this as a poor-quality or problematic question

A 72-year-old man presents with 4 weeks of shortness of breath (SOB), shoulder pain, weight loss, cervical lymphadenopathy, and finger clubbing. What is the likely diagnosis?

Mark this as a high-quality question
Mark this as a poor-quality or problematic question

A 66-year-old man, active with a good diet, whose mother had osteoporosis at age 91, wants to check his osteoporosis risk. What risk factors prompt a bone mineral density (BMD) test?

Mark this as a high-quality question
Mark this as a poor-quality or problematic question

A 72-year-old man with a history of a reducible inguinal hernia presents to the emergency department with severe groin pain and a firm, tender mass in the right inguinal region. He reports that he has been unable to reduce the hernia despite his usual maneuvers. He also reports nausea and vomiting for the past 6 hours. On examination, his vital signs are stable, but he appears uncomfortable. The mass in his groin is erythematous and exquisitely tender to palpation. Which of the following is the most appropriate next step in management?

Mark this as a high-quality question
Mark this as a poor-quality or problematic question
Right leg affected by warfarin necrosis
Image by Bakoyiannis C, Karaolanis G, Patelis N, Maskanakis A, Tsaples G, Klonaris C, Georgopoulos S, Liakakos T CC BY 4.0 · Source

A 75-year-old woman on warfarin for chronic atrial fibrillation presents to the emergency department with a 24-hour history of rapidly worsening pain and discolouration in her right lower leg. She denies trauma. Her INR is 2.8. Physical examination reveals the findings shown in the image. Peripheral pulses are palpable. What is the most appropriate immediate management step?

Mark this as a high-quality question
Mark this as a poor-quality or problematic question
feedback